Vous êtes sur la page 1sur 5

Antigonal conjugate of I wrt its cevian triangle lies on O'I

Let

be

intersections of

triangle
,

antigonal conjugate of
where

and

let

be

its

incenter.

with the sides

Let

, and

with respect to triangle

is the circumcenter of triangle

be

the

. Prove that the


lies on the line

Solution 1.- Hope this will be helpful


Rewrite

as

Let

be the projection of

on

Let

be the projection of

on

Let

be the midpoint of

Invert

with

respect

to

and

.
.
.

denote

as

inversion.Easy to see
of

all lie on
on

on

Since

harmonic quadrilateral. ie.


Similarly, we can get
From the definition of

so

of

and

the projection of

Since

image

under

this

is the midpoint

Since

of

the

,so

. Similarly, we can get

is the midpoint of

through

so

is

is the midpoint of
we can

is

is the projection of
,

with respect to
pass

are collinear and

.
get

is the reflection

.
the

is the nine point circle of triangle

midpoint
. ie.

of

is the nine point center

of triangle

. Since the Antigonal conjugate of

the

Poncelet

point

is the reflection of

of

and

lie

in
on

,
so we get the image of the Antigonal conjugate of
and

is the midpoint of

is the common point of

which is the

Anti-steiner point of Euler line (called Kiepert focus which is


triangle
the

in ETC ) of

. (I use a property of Anti-steiner point here: If


reflection

of

with

respect

to

is
,

then

concur at the Anti-steiner point of the


line pass through
triangle
on

the

and the orthocenter of triangle

). Since
line

pass

is the pedal circle of

through

and

the

Kosnita

with respect to
, so the image of

point

of

lie

triangle

( Kosnita point is the isogonal conjugate of the nine point center which is
in ETC ). Now we only have the show

are collinear ... this

is a well known fact but I don't have the synthetic proof yet
Comment:

Now

we

only

have

to

prove

Theorem:
Let

be the nine point center of

Let

be the Kosnita point of

Let

be the Kiepert focus of

Then

are collinear

the

following

theorem

Proof:
Let

be the circumcenter of

Let

be the orthocenter of

Let

be the tangential triangle of

Let

be the midpoint of

Let

be the reflection of

Since

, respectively .
in

point

of

of

factor

of

the

circumcenter

then
of

under this homothetic is

.
is the incenter of

(Antigonal

get

is

is the

. Consider the homothetic with center

hence the image

so

Since

, so

, respectively . So from the property of the Kosnita

point we get

Since

, respectively .

is the Anti-steiner point of the Euler line of

Feuerbach

and

conjugate

of

, so the image
the

incenter).

Since

of
is

from http://www.artofproblemsolving.com/Foru
are collinear, hence

is
the
...

of
of

6&t=612271 we

are collinear .

Q.E.D
Problem of conic and three circles!
Let
Let

are concyclic;

are concyclic;

are on conic. Prove that:

are concyclic.
are concyclic.

Answer.given

Label
conic

axes

the
with

Arbitrary

circle w

through A,

D cuts C again at
are points on
such that the tangents
of
to

respectively.

and

By

Hence,
when

is either on

Let

generalized

or

varies, keeping

the

intersection
power

and
fixed, all lines

As a result, we deduce that


at

be

of

at
of

these

point,

tangents

we

are equal inclined to

have

Thus

go through a fixed direction.

and

then

If
are concyclic.

An inequality in a square
I research a pure geometric solution without any calculation
1. ABCD a square
2. E a point on the segment AD
3. F the foot of the perpendicular to CE through B
4. M the midpoint of EF

are parallel

cuts

again

5. G the point of intersection of the parallel to BC through M with the


perpendicular bissector of BF.
Prove : AC < 2.FG.
Sincerely, Jean-Louis
Answer.- Courtesy of Clarence Chew:
We want to show that: 2BC2=AC2<4FG2=4FX2+4XG2=BF2+4XG2 (let projection
of G to BF be X)
If we draw a line parallel to BC at X intersecting CF at Y, then Y is midpoint of
CF. XYMG is parallelogram, hence XG=XY=CE/2.
So

we

have

CF2<ED2.

to
Using

show

2BC2<BF2+CE2.
similarity

Using

of

Pythagorean
BFC

and

theorem,
CDE,

CF/ED=BC/CE=CD/CE. Now the result is obvious by hypotenuse longest side of


triangle.
Answer 2.1. C* the point of intersection of CE and BG.
2. FC = EC*3. according to Pythagoras theorem applied to FBC* and DAC, we
are done (a + 1/a >= 2 with a strictly positive)
Sincerely, Jean-Louis

Vous aimerez peut-être aussi